Vector g is shown below.
Vector h is parallel to g, in the same direction and
half as long.
Work out h as a column vector.
9

Vector G Is Shown Below.Vector H Is Parallel To G, In The Same Direction Andhalf As Long.Work Out H As

Answers

Answer 1

Answer:

h = [tex]\left[\begin{array}{ccc}5\\1\\\end{array}\right][/tex]

Step-by-step explanation:

g = [tex]\left[\begin{array}{ccc}10\\2\\\end{array}\right][/tex] , then

h = [tex]\frac{1}{2}[/tex] [tex]\left[\begin{array}{ccc}10\\2\\\end{array}\right][/tex] = [tex]\left[\begin{array}{ccc}5\\1\\\end{array}\right][/tex]


Related Questions

A woman passed gas silently. I said "it stinks", and she said "I apologize. Excuse me". Why did she say both of those things?

Answers

She says “I apologise” because she feels bad that she made a bad smell and she says “excuse me” because she wants you to accept her apology

How many pounds of candy that sells for ​$0.82 per lb must be mixed with candy that sells for ​$1.36 per lb to obtain 9 lb of a mixture that should sell for ​$0.91 per​ lb?

Answers

7.5 pounds of the $0.82 per lb candy must be used in the mixture.

How many pounds of each candy should we use?

First, let's define the variables:

x = pounds of the $0.82 candy used.y = pounds of the $1.36 candy used.

We want to make 9 lb of mixture, then:

x + y = 9.

And the price of these 9 pounds must be $0.91, then we can write:

x*$0.82 + y*$1.36 = 9*$0.91 = $8.19

Then we have a system of equations:

x + y = 9.

x*$0.82 + y*$1.36 = $8.19

We can isolate y on the first equation so we get:

y = 9 - x

Now we can replace that on the other equation:

x*$0.82 + (9 - x)*$1.36 = $8.19

And now we can solve this for x.

x*($0.82 - $1.36) = $8.19 - 9*$1.36

-x*$0.54 = -$4.05

x = (4.05/0.54) = 7.5

So 7.5 pounds of the $0.82 per lb candy must be used in the mixture.

If you want to learn more about systems of equations:

https://brainly.com/question/13729904

#SPJ1

|4x + 7| − 4 = 20
Can anyone help me with this

Answers

Answer:

x = -31/4  or x = 17/4

Step-by-step explanation:

|4x + 7| − 4 = 20

⇔ |4x + 7| − 4 + 4 = 20 + 4

⇔ |4x + 7| = 24

⇔ 4x + 7 = 24  or 4x + 7 = -24

⇔ 4x = 24 - 7 or 4x = -24 - 7

⇔ 4x = 17 or 4x = -31

⇔ x = 17/4 or x = -31/4

Answer:

x = [tex]\frac{17}{4}[/tex]    (17/4 = 4.25)

or x =  [tex]-\frac{31}{4}[/tex]  (-31/4 = -7.75)

Step-by-step explanation:

| | is notation for absolute value

absolute value - the distance that a number is from 0

> essentially, you can think of absolute value as the "positive version" of whatever is inside of the | |

if we have | x | = 20, we could really have (without the | | ) two versions of x

either | x | = 20  ; or | -x | = 20  {because a negative x inside of the | | has the same value as positive x}

we set this up as two equations:

x = 20        or         -x = 20

                                ^ {multiply by -1}

x = 20  or   x = -20

now, let's plug our understanding into the equation

|4x + 7| - 4 = 20

first, we should simplify our equation to:

|4x + 7| - 4 = 20

           + 4   + 4

 |4x + 7|   =   24

now, let's separate this absolute value equation into two separate equations:

4x + 7 = 24:

      - 7    - 7  {subtract 7 from both sides to isolate x}

4x = 17

÷4  ÷4        {divide both sides by 4 to get 1x}

x= [tex]\frac{17}{4}[/tex]

or,

4x + 7 = -24:

      - 7    -7   {subtract 7 from both sides to isolate x}

4x     =    -31

÷4           ÷4   {divide both sides by 4 to get 1x}

 x = [tex]-\frac{31}{4}[/tex]

so, we know that

x = [tex]\frac{17}{4}[/tex]    (17/4 = 4.25)

or x =  [tex]-\frac{31}{4}[/tex]  (-31/4 = -7.75)

hope this helps!!

Find an equation for the line that passes through the point P(-5,-3) and is parallel to the line
7x + 4y
10. Use exact values.

Answers

-------------------------------------------------------------------------------------------------------------

Answer:  [tex]\textsf{y = -1.75x - 11.75}[/tex]

-------------------------------------------------------------------------------------------------------------

Given:  [tex]\textsf{Goes through (-5, -3) and parallel to 7x + 4y = 10}[/tex]

Find:  [tex]\textsf{The equation in slope-intercept form}[/tex]

Solution: We need to first solve for y in the equation that was provided so we can determine the slope.  Then we plug in the values into the point-slope form, distribute, simplify, and solve for y to get our final equation.

Subtract 7x from both sides

[tex]\textsf{7x - 7x + 4y = 10 - 7x}[/tex][tex]\textsf{4y = 10 - 7x}[/tex]

Divide both sides by 4

[tex]\textsf{4y/4 = (10 - 7x)/4}[/tex][tex]\textsf{y = (10 - 7x)/4}[/tex][tex]\textsf{y = 10/4 - 7x/4}[/tex][tex]\textsf{y = 2.5 - 1.75x}[/tex]

Plug in the values

[tex]\textsf{y - y}_1\textsf{ = m(x - x}_1\textsf{)}[/tex][tex]\textsf{y - (-3) = -1.75(x - (-5))}[/tex]

Simplify and distribute

[tex]\textsf{y + 3 = -1.75(x + 5)}[/tex][tex]\textsf{y + 3 = (-1.75 * x) + (-1.75 * 5)}[/tex][tex]\textsf{y + 3 = -1.75x - 8.75}[/tex]

Subtract 3 from both sides

[tex]\textsf{y + 3 - 3 = -1.75x - 8.75 - 3}[/tex][tex]\textsf{y = -1.75x - 8.75 - 3}[/tex][tex]\textsf{y = -1.75x - 11.75}[/tex]

Therefore, the final equation in slope-intercept form that follows the information that was provided is y = -1.75x - 11.75

Find the range of the function.
f(x) = 10-x²
a. [5,00)
c. (-∞0, 10]
b. (-∞, -1) U [0, 00)
d.
Please select the best answer from the choices provided
(-∞, -1) U [∞o, 10)

Answers

Answer: hard to tell but I think D is correct (−∞,10],{y| y ≤ 10}

Step-by-step explanation:

Range:

(−∞,10] or {y| y ≤ 10}

because there is a vertex at (0, 10)

(found this by using -b/2a giving us 0 for x coord and plugging in 0 to 10-x² giving us (0, 10)

Then understanding since -x² is negative, the parabola goes down. And we know that the range is anything less or equal to 10

The Strikers soccer team has 20 members, and 8 of them play offense. What percent of the team members play offense?

Answers

Answer:

40%

Step-by-step explanation:

We already have our first value 20 and the second value 8. Let's assume the unknown value is Y which answer we will find out.

As we have all the required values we need, Now we can put them in a simple mathematical formula as below:

Step 1 ⇒ Y = 8/20

By multiplying both numerator and denominator by 100 we will get:

Step 2 ⇒ Y = 8/20 × 100/100 = 40/100

Step 3 ⇒ Y = 40

Finally, we have found the value of Y which is 40 and that is our answer.

To purchase 13700 worth of restaurant equipment for her business Maria made a down payment of 1500 and took out a business loan for the rest after 3 years of paying monthly payments of 371.16 she finally paid off the loan
What was the total amount Maria ended up paying for the equipment

How much internet did Maria pay on the loan

Answers

The total amount Maria ended up paying for the equipment will be $14,861.76. And The interest of Maria on the loan will be 8.48%.

What is Algebra?

The analysis of mathematical representations is algebra, and the handling of those symbols is logic.

To purchase 13700 worth of restaurant equipment for her business.

Maria made a down payment of 1500 and took out a business loan for the rest, after 3 years of paying monthly payments of 371.16 she finally paid off the loan.

The total amount Maria ended up paying for the equipment will be

Total amount = 371.16 × 3 × 12 + 1500

Total amount = $14,861.76

The interest of Maria on the loan will be

Interset = [(14861.76 – 13700) / 13700] x 100

Interset = 8.48%

More about the Algebra link is given below.

https://brainly.com/question/953809

#SPJ1

(7 + 7i)(2 − 2i)
(a) Write the trigonometric forms of the complex numbers. (Let
0 ≤ < 2.)

(7 + 7i) =

(2 − 2i) =



(b) Perform the indicated operation using the trigonometric forms. (Let
0 ≤ < 2.)



(c) Perform the indicated operation using the standard forms, and check your result with that of part (b).

Answers

The complex number  -7i into trigonometric form is 7 (cos (90) + sin (90) i) and  3 + 3i in trigonometric form is 4.2426 (cos (45) + sin (45) i)

What is a complex number?

It is defined as the number which can be written as x+iy where x is the real number or real part of the complex number and y is the imaginary part of the complex number and i is the iota which is nothing but a square root of -1.

We have a complex number shown in the picture:

-7i(3 + 3i)

= -7i

In trigonometric form:

z = 7 (cos (90) + sin (90) i)

= 3 + 3i

z = 4.2426 (cos (45) + sin (45) i)

[tex]\rm 7\:\left(cos\:\left(90\right)\:+\:sin\:\left(90\right)\:i\right)4.2426\:\left(cos\:\left(45\right)\:+\:sin\:\left(45\right)\:i\right)[/tex]

[tex]\rm =7\left(\cos \left(\dfrac{\pi }{2}\right)+\sin \left(\dfrac{\pi }{2}\right)i\right)\cdot \:4.2426\left(\cos \left(\dfrac{\pi }{4}\right)+\sin \left(\dfrac{\pi }{4}\right)i\right)[/tex]

[tex]\rm 7\cdot \dfrac{21213}{5000}e^{i\dfrac{\pi }{2}}e^{i\dfrac{\pi }{4}}[/tex]

[tex]\rm =\dfrac{148491\left(-1\right)^{\dfrac{3}{4}}}{5000}[/tex]

=21-21i

After converting into the exponential form:

[tex]\rm =\dfrac{148491\left(-1\right)^{\dfrac{3}{4}}}{5000}[/tex]

From part (b) and part (c) both results are the same.

Thus, the complex number  -7i into trigonometric form is 7 (cos (90) + sin (90) i) and  3 + 3i in trigonometric form is 4.2426 (cos (45) + sin (45) i)

Learn more about the complex number here:

brainly.com/question/10251853

#SPJ1

find the slope of the line that passes through (3,10) and (1,17)​

Answers

Answer:

[tex]m=- \frac{7}{2}[/tex]

Step-by-step explanation:

The slope of a line passing through the two points [tex]\displaystyle{\large{{P}={\left({x}_{{1}},{y}_{{1}}\right)}}}[/tex] and[tex]\displaystyle{\large{{Q}={\left({x}_{{2}},{y}_{{2}}\right)}}}[/tex] is given by [tex]\displaystyle{\large{{m}=\frac{{{y}_{{2}}-{y}_{{1}}}}{{{x}_{{2}}-{x}_{{1}}}}}}[/tex].

We have that [tex]x_1=3[/tex], [tex]y_1=10[/tex], [tex]x_2=1[/tex], [tex]y_2=17[/tex].

Plug the given values into the formula for slope: [tex]m=\frac{\left(17\right)-\left(10\right)}{\left(1\right)-\left(3\right)}=\frac{7}{-2}=- \frac{7}{2}[/tex]

Answer: the slope of the line is [tex]m=- \frac{7}{2}[/tex].

Answer:

slope = - [tex]\frac{7}{2}[/tex]

Step-by-step explanation:

calculate the slope m using the slope formula

m = [tex]\frac{x_{2}-y_{1} }{x_{2}-x_{1} }[/tex]

with (x₁, y₁ ) = (3, 10 ) and (x₂, y₂ ) = (1, 17 )

m = [tex]\frac{17-10}{1-3}[/tex] = [tex]\frac{7}{-2}[/tex] = - [tex]\frac{7}{2}[/tex]

Need help with the following question

Answers

3 times. Draw a line 1 unit up, and you can see that there are 3 intersections.

Which of the following terms best fits this definition?

The angle between two sides of a triangle.

Select one:

AAS Theorem


SAS Postulate


Included Side


ASA Postulate


HL Congruence Theorem


Included Angle


SSS Postulate

Answers

Step-by-step explanation:

hope you can understand

I need help ASAP please

Which statement is true about whether Z and B are independent events?

A: Z and B are independent events because P(Z | B) =P(Z).
B: Z and B are independent events because P(Z | B) = P(B).
C: Z and B are not independent events because P(Z | B) ≠ P(Z).
D: Z and B are not independent events because P(Z | B) ≠ P(B).

Answers

A

Explanation:

Independent event means P(Z|B)=P(Z)

P(Z|B)=(Z ∩ B)/B
=(126)/280
=.45

P(Z)=297/660
=.45

Therefore they are independent events, and the answer is A.
The answer should be Option A

Which of the following best describes the expression below when i = √-1?
3+4i
A. Complex number
B. Real number
C. Irrational number
D. Rational number

Answers

The answer is A. Complex number

What are the solutions to the equations y= 4x^2+5x-6

Answers

Answer:

D. x = -2; x = 3/4

Step-by-step explanation:

Read the values of a, b, and c from the quadratic equation: a is the number in front of x^2, b is the number in front of x, c is the number at the end. In our case: a = 4,b = 5,c = −6 The formula for the roots is =  [tex]\frac{-b +-\sqrt{b^{2}-4ac } }{2a\\}[/tex]

A number is equal to the sum of half a second number and 3. The first number is also equal to the sum of one- quarter of the second number and 5. The situation can be represented by using the graph below, where x represents the second number. 16 NO 8 K 3₂ 5 2 4 st 6 8 10 12 14 16 Which equations represent the situation? Hurry ​

Answers

5 1/2
this is how we do it, jus learn abt it

A woman passed gas silently. I said "it stinks", and she said "I apologize. Excuse me". Why did she say both of those things?

Answers

Answer:

one possible way was because she knew what she did and started playing innocent

Find the area of the trapezoid. TOP 11ft, RIGHT4√3ft , BOTTOM 15ft ,LEFT 8ft

Answers

Answer:

[tex]52\sqrt{3} ft^{2}[/tex]

Step-by-step explanation:

Please refer to the attached picture.

First we will find the area of rectangle BCDE.

Area of Rectangle = Length x Breadth = DE x CD

= 11 x [tex]4\sqrt{3}[/tex]

[tex]=44\sqrt{3} ft^{2}[/tex]

Next we will find Area of Triangle ABE.

Area of Triangle = 0.5 x Base x Height

[tex]0.5*4*4\sqrt{3} \\=8\sqrt{3} ft^{2}[/tex]

Area of Trapezoid = Area of Rectangle + Area of Triangle

[tex]=44\sqrt{3} +8\sqrt{3} \\=52\sqrt{3} ft^{2}[/tex]

Answer:

A = 52[tex]\sqrt{3}[/tex] ft² ≈ 90.1 ft²

Step-by-step explanation:

the area (A) of a trapezoid is calculated as

A = [tex]\frac{1}{2}[/tex] h (b₁ + b₂ )

where h is the perpendicular height and b₁ , b₂ the parallel bases

here h = 4[tex]\sqrt{3}[/tex] , b₁ = 15 , b₂ = 11 , then

A = [tex]\frac{1}{2}[/tex] × 4[tex]\sqrt{3}[/tex] × (15 + 11)

   = 2[tex]\sqrt{3}[/tex] × 26

   = 52[tex]\sqrt{3}[/tex] ft²

   ≈ 90.1 ft² ( to the nearest tenth )

can someone help me asap :)

Answers

Answer:

Option 2 - TU is perpendicular to MN and MN is parallel to PQ

angle M = angle P ( 90°)

so MN is parallel to pq

MN makes 90° with UT so it is perpendicular

what is the ratio of the radius of circle a to the radius of circle b?

Answers

Answer:

3:1

Step-by-step explanation:

Assuming that the edges of the circle are supposed to line up with the dotted lines of the graph, all you have to do is count how many lines are between the middle of the circle and the edge (either directly vertical or horizontal due to the graph we are using). We can see that the radius of circle A is approximately 3 units, while the radius of circle B is approximately 1 unit. So the ratio of the radius of circle A to the radius of circle B is 3:1.

Answer:

3:1

Step-by-step explanation:

because once count the radius of the big circle that is 3 unit and the radius of small circle is 1 unit

A rectangular parking lot has a perimeter of 820 ft. The area of the parking lot measures 42,000 ft2. What is a dimension of the parking lot?

Answers

By solving a system of equations, we will see that the parking lot is 210ft by 200ft.

How to get the dimensions of the parking lot?

For a rectangle of length L and width W, the perimeter is:

P = 2*(L + W)

And the area is:

A = L*W

Here we know that the perimeter is 820 ft and the area is 42,000 ft²

Then we can write the two equations (ignoring units).

820 = 2*(L + W)

42,000 = L*W

We can isolate L in the first equation to get:

820/2 = L + W

410 - W = L

Now we can replace that in the other equation:

42,000 = (410 - W)*W = 410*W - W^2

Now we want to solve the quadratic equation:

-W^2 + 410*W - 42,000 = 0

The solutions are given by:

[tex]W = \frac{-410 \pm \sqrt{410^2 - 4*(-1)*(-42000)} }{-2} \\\\W = \frac{-410 \pm 10 }{-2}[/tex]

Then the solutions are:

W = (-410 + 10)/(-2) = 200

W = (-410 - 10)/2 = 210

If we take W = 200, then:

L = 410 - W  = 410 - 200 = 210

So we can conclude that the parking lot is 200ft by 210ft.

If you want to learn more about rectangles:

https://brainly.com/question/17297081

#SPJ1

Answer: C, 210 ft

Step-by-step explanation: edge :)

four big water bottles can hold 8 gallaons how much can 10 big water gallons hold

Answers

4 can hold 8 gallons = 8/4 = 2 big bottle per gallons.

For 10 big bottles = 10 x 2 = 20 gallons.

If you are looking for how many bottles can be filled with 10 gallons, the answer is 5.
Let x represent the gallons you are finding
If 4=8 then,
10=x
You cross multiply
4x = 10*8
4x /4= 80/4
x = 20

1
2
3
5
10
Two runners are saving money to attend a marathon.
The first runner has $110 in savings, received a $45 gift
from a friend, and will save $25 each month. The
second runner has $50 in savings and will save $60
each month.
After how many months will both runners have the
same amount of money?
02
O 3

Answers

Answer:

3 months

Step-by-step explanation:

The first runner: $110 + $45 = $155 starting out, plus 25x for $25 each month.

The second runner: $50 starting out, plus 60x for $60 each month.

To find out when both runners have the same amount of money, we will set the expressions equal to each other and solve.

155 + 25x = 60x + 50

105 = 35x

x = 3

Brainliest, please :)

What are m and b in the linear equation y=16+6x

Answers

Answer: B=16 and m=6

Step-by-step explanation:

in y=mx+b the number next to x is always the m/slope and the number without a variable is always the b.

Create and solve a linear equation that represents the model, where circles and a square are shown evenly balanced on a balance beam.

Answers

A linear equation that represents the model is: x + 6 = 10; x = 4

How to solve linear equations?

Let us first define the variables based on the attached image of the ball balance:

Let x = number of balls that contains the square.

On the left side, we have; square + 6 balls

On the right side, we have; 10 balls

To balance this, we have;

x + 6 = 10

x = 10-6

x = 4

Thus, a linear equation that represents the model is:

x + 6 = 10; x = 4

Read more about Linear equations at; https://brainly.com/question/9406333

#SPJ1

One positive number is 8 times another number. Their difference is 70.
Which of the following equations could be used to find the numbers?

Answers

Answer:

X equals 10.

8x - x = 70

Step-by-step explanation:

8 x 10 = 80

80 - 10 = 70

!!!!!!!!!!!!!!!!!!!!!!!!!!!!!!!!!!!!!!!!

Answers

[tex]\quad \huge \quad \quad \boxed{ \tt \:Answer }[/tex]

[tex] \texttt{ \:The absolute maxima of f is f(-8) = 6} [/tex]

____________________________________

[tex] \large \tt Solution \: : [/tex]

Absolute maxima is the maximum possible value for a given x, of a function.

and here, the maximum value is at -8, and the maximum value is 6.

[tex]\qquad \tt \rightarrow \: maximum - \: \: f( - 8) = 6[/tex]

Answered by : ❝ AǫᴜᴀWɪᴢ ❞

A hyperbola centered at (7, 0) has a focus at (7, 5) and vertex at (7, 4). Which is the equation of the hyperbola in standard form?

quantity x minus 7 end quantity squared over 16 minus y squared over 9 equals 1
quantity x minus 7 end quantity squared over 25 minus y squared over 16 equals 1
y squared over 16 minus quantity x minus 7 end quantity squared over 9 equals 1
y squared over 25 minus quantity x minus 7 end quantity squared over 16 equals 1

Answers

Based on the calculations, the equation of this hyperbola in standard form is: A. [tex]\frac{x\;-\;7}{16} + \frac{y}{9} = 1[/tex].

How to determine the equation of a hyperbola?

Mathematically, the equation of a hyperbola in standard form is given by:

[tex]\frac{x\;-\;h}{a^2} + \frac{x\;-\;k}{b^2} = 1[/tex]

Given the following data:

Center (h, k) = (7, 0)

Vertex (h+a, k) = (7, 4)

Focus = (h+c, k) = (7, 5)

Also, we can deduce that the value of a and c are 4 and 5 respectively.

For the value of b, we would apply Pythagorean's theorem:

c² = a² + b²

b² = c² - a²

b² = 5² - 4²

b² = 9.

Substituting the parameters into the standard equation, we have:

[tex]\frac{x\;-\;7}{4^2} + \frac{y\;-\;0}{3^2} = 1\\\\\frac{x\;-\;7}{16} + \frac{y}{9} = 1[/tex]

Read more on hyperbola here: https://brainly.com/question/3405939

#SPJ1

Select the correct answer.
x

f(x)

2.0 2.8
2.5 1.1
3.0 –0.8
3.5 –1.2
4.0 –0.3
4.5 0.7
For the given table of values for a polynomial function, where must the zeros of the function lie?

A.
between 2.0 and 2.5 and between 4.0 and 4.5

B.
between 2.5 and 3.0 and between 4.0 and 4.5

C.
between 2.0 and 2.5 and between 3.5 and 4.0

D.
between 2.5 and 3.0 and between 3.5 and 4.0

Answers

Polynomials are mathematical expressions involving variables raised with non-negative integers and coefficients. The correct option is B.

What is a polynomial?

Polynomials are mathematical expressions involving variables raised with non-negative integers and coefficients(constants who are in multiplication with those variables) and constants with only operations of addition, subtraction, multiplication, and non-negative exponentiation of variables involved.

Example:

 x² + 3x + 5

In order to find the values at which the given polynomial will have zeros of the function, we need to find the values at which f(x) changes from positive to negative or vice versa. Since this is the range at which the function must have crossed the x-axis on the graph.

As per the given table, the value of f(x) is changing from negative to positive and positive to negative are between 2.5 and 3.0 and between 4.0 and 4.5.

Hence, the correct option is B.

Learn more about Polynomials:

https://brainly.com/question/27343162

#SPJ1

Question 10 of 10
Rewrite the following linear equation in slope-intercept form. Write your
answer with no spaces.
v+2=4(x-3)
Answer here

Answers

y+2=4(x-3)
y+2=4x-12
y+2-2=4x-12-2
y=4x-14

March 8, 2017, one U.S. dollar was worth 66.79 Indian rupees.
a) On that date, how many dollars was 110.66 rupees worth?
Round your answer to the nearest hundredth of a dollar. I need help with this question.

Answers

[tex] \huge \tt \underline {\green{Answer}}[/tex]

If on March 8, 2017 , one U.S. dollar worth 66.79 Indian rupees

ie. $1 = Rs 66.79

$ 1 = 66.79 × 1

$ ? = 110.66

$ = New / old

$ = 110.66 / 66.79

$ = 1.65683485552

or

$1.66 = 110.66

Other Questions
What is the slope of a line that is perpendicular to the line y = -1/2x+5-2-1/21/22 Calculate this logarithmic expression 16-2t=3/2t+9 solve for t Towering bulwarks are popular rides at amusement parks.True or False Select the correct answer. What is the best conclusion to be drawn from this passage? QA Choosing a college can be a very stressful time for students. OB. Students should consider only financial aid when choosing a college. OC Students should make an informed decision when choosing a college. OP. Choosing a college can be a very exciting time for students Reset Next Select the correct answer . What is the best conclusion to be drawn from this passage ? QA Choosing a college can be a very stressful time for students . OB . Students should consider only financial aid when choosing a college . OC Students should make an informed decision when choosing a college . OP . Choosing a college can be a very exciting time for students Reset Next rrrrrrrrrrrrrrrrrrrrr The average atomic mass of carbon is 12.01 amu. Based on the atomicmasses of the two isotopes of carbon, how do the relative abundances of theisotopes compare?IsotopeAtomic mass (amu)C-1212.000C-1313.003OA. There is a slightly larger percentage of C-12 than C-13.OB. There is a very large percentage of C-12.OC. There is a very small percentage of C-12.OD. They are about the same. Evaluate the integral-7xx +1Note: Use an upper-case "C" for the constant of integration.Idx Due to the culture of the colonists in North America, what religion did manyAfrican slaves adopt? A. IslamB. Christianity C. PuritanismD. Wolof please help! I need this quicklywrite an equation in standard form of the line passing through the points (12,6) and (-3,11) what was one of the first thing that the Brain Trust told FDR he needed to do Which most likely describes a person with low self-esteem? Select the correct answer.Identify the end behavior and the zeros of function h.h(1) = -1 - 91 +41 +96Based on these key features, which statement is true about the graph representing function h? A.The graph is negative on the intervals (- infinity, -8) and (-4, 3).B. The graph is positive on the intervals (-8, -4) and (3, infinity).OC. The graph is negative on the intervals (-3, 4) and (8, ).D. The graph is positive on the intervals (-infinity ,-8) and (-4, 3). Type the correct answer in each box. Use numerals instead of words. If necessary, use / for the fraction bar(s). Answer? Having trouble understanding this question It was reported that uninsured drivers caused 1 in 20 accidents. About 682,494 car crashes were reported. About how many of these drivers were uninsured? In the equation y/6 = 156, what is the next step in the equation solving sequence Why did the Committee of Merchants for the Relief of Colored People Suffering from the Late Riots emphasize that Peter Heuston's "wife died about three weeks before the riots, leaving with her husband an only child... aged eight years"? atal Hyk was a center for deity worship.True or false Robert Hooke first observed cork under a microscope in jail so he named the small cubicles cells False True